- Joined
- Jan 2025
- Subscription
- Core
Am I the only one who, after reading the example, automatically thought about:
Domain: Resident
Rule: /Purpose → Prohibit
I just imagined a building with all the residents (hence the domain), and within that building, we’re discussing the rule (/Purpose → Prohibit). Did anyone else think of it this way, or am I missing something crucial?
I understood the lesson, but I found my initial reasoning easier to visualize and comprehend, at least for me.
Cheers from France,
Benjamin
This is so frustrating. I was thinking exactly like J.Y. during the timed PT. I even thought, “Yeah, this doesn’t fully justify the contention, that’s not it,” and ended up choosing another answer choice that had nothing to do with the contention.
That said, I had a different interpretation of D than J.Y. did. I eliminated D because I thought: Just because you can drill wells much deeper than we do today doesn’t mean you’ll actually find more reservoirs. What if there simply aren’t any deeper down? So I was like, “Get out of here, D.”
But then again, the question stem said “MOST helps,” so D is still the best of the five answer choices, even though it still feels very weak to me.
I'm not sure I understand why "jurors are usually eager to SOLIDIFY their position" (in the text) is taken to mean "to arrive at a FIRM view." (in the answer choice D).
Given that the question stem asks 'which one of the following does the author mention', we're not dealing with a Most Strongly Supported (MSS) question or looking for the 'best' inference. We're looking for something explicitly stated. Assuming that to 'solidify' a position necessarily means to 'arrive at a firm view' seems unreasonable to me.
For example, suppose I'm only 10% confident in my view, and the stealing thunder strategy helps me reach 30%. I’ve strengthened—or solidified—my view, but that doesn't mean I’ve arrived at a firm conclusion. It just means my view has become more solid than it was before.
While I understand why D is considered correct compared to the other four answer choices, I’m not convinced that D is 100% correct on its own, and regarding what the question stem is asking us.
Does anyone see a flaw in my reasoning?
I am SO dumb. I think I just made the worst unwarranted assumption of my life. I chose (C) ‘Unlike ocean stickleback, the lake stickleback are more often preyed upon by predatory insects than by larger fish.’
My reasoning at the time was: ‘Yeah, they’re larger, so insect predators have more surface area to attack, which means insects have a higher chance of getting to them. So being larger doesn’t help protect them from predators, in fact, it makes them weaker.’
Hahahaha. I have no idea what was going on in my head when I picked that.
I’d bet my future 170+ LSAT score that, during LSAC’s question-writing meetings, someone considered including the answer choice “pee.”
I got this one wrong because I thought every novel had only one genre. So, when the author says the novel is a “great dramatic novel,” I was like, “Nice. you think it’s a drama, so it would make sense for you to say that classifying it as science fiction is wrong. I get you.” And if you assume, like I did, that a novel can only have one genre, then I believe answer choice A would be sufficient. But because a novel can have multiple genres, answer choice A is not sufficient.
Hello to all my 7Sage comrades,
I just noticed this question was posted at the very beginning of the curriculum, and I felt compelled to write this message.
This is my second (or third) time going through the entire curriculum, so I understand that some (or many) of you might feel overwhelmed when facing this question for the first time. You might even feel tempted to leave a comment expressing your frustration. Trust me, I have been there. But let me reassure you: I wholeheartedly agree with J.Y. on this one—I wish every LSAT question were like this.
If this is your first time encountering this question and you chose the wrong answer, I want you to know that it is completely fine. You have only just started your LSAT preparation! As you work through the curriculum and learn the foundational concepts, you will come to see why this question is purely mechanical and not as complicated as it seems (once you know the tricks you will learn here).
Stay motivated. Easier said than done, right? There will be happy moments during your LSAT studies, but also moments when you will feel stuck, or even like quitting. While that would mean less competition for me (a little sarcasm never hurts), please do not give up! By sticking with the curriculum, you will experience plenty of “ah-ha!” moments where concepts finally click. Listen carefully to J.Y.’s explanations—even when they feel long—because his in-depth analysis and extra exercises are invaluable. Trust his method; it works (and no, 7Sage does not pay me to say that).
When I started, my diagnostic score was 150—not terrible, but not great. With 7Sage's help, I scored 160 (75th percentile) on my first LSAT, and 166 (90th percentile) on my second. English is not my first language, so if I could achieve this, I know you can too. I am preparing to take the LSAT for a third time to aim even higher, and I owe my progress to this curriculum and consistent practice.
My advice? Truly understand the curriculum—do not just skim it. Take Practice Tests, do the Blind Review (you will learn about this soon), and embrace every mistake as a learning opportunity. Practice is the real secret to mastering the LSAT.
To all my comrades, even though you are technically my competition, I sincerely wish you the best in your LSAT journey. Do not get discouraged, do not convince yourself you are “bad” at this, and most importantly, do not quit. Nearly everyone faces moments of doubt. Push through them, learn from every challenge, and keep practicing. If you can read this right now, it means you are already on your way to master this LSAT.
Good luck to all of you, and I hope to see you on the benches of one of the best law schools soon.
Best,
Benjamin
I am French, and I approve fat cats singing French lullabies from the Belle Époque. Meow Meow.
Alright. So got E in the first place but even after the explanation, I understand why A is correct, but I still don't understand why E is incorrect.
From the stimulus, I got that :
Manners --> Social
Morals --CAN BE--> Social (therefore can be "alone" as well)
I chose E because I made the following connection :
Answer E "What is social in nature could not be a matter of morality" = I thought this would be true because what is social in nature, if it is not a matter of morality (even though it can be, according to the stimulus) could be a matter of manners.
So, I guess my equation does not work. I, however, don't understand why. If someone can enlighten me, that would be awesome.
Cheers from France.
Benjamin
Even though I carefully analyzed everything in that question, A being incorrect requires us to make an assumption, which is...bad (?). In fact, even if the soft-drink industry as a whole experienced depressed sales during summer months, that absolutely DOES NOT mean why sales of our particular Foamy Soda went down. If we think A is a viable reason to explain the discrepancy, it requires us to make the assumption that because the whole went down, our Foamy Soda went down too, which is typically a reasoning flaw we learnt to avoid. Am I the only one seing that? #help #feedback
Square, circle, triangle... J.Y. is the new Front Man.
Sharing my tactic here: I quickly eliminated D because it says, 'ANY substance... WILL eventually BE DISCOVERED.' However, based on the stimulus, it's not necessary for every substance to be discovered. Even if just some are discovered, the argument could still hold.
As an English-as-a-second-language test taker, I found this vocabulary question quite tough. But by sticking to the “lawgic,” we can still get it right, even without fully knowing the meaning of a word. For those of us taking the LSAT in a second language: keep going! You can do it!
#help #feedback
Does anyone else agree and am I the only one to see that:
- For (A), it is a possibility that, with elastic bands, astronauts could do weight-bearing exercices, therefore making (A) the correct answer choice (not strenghtening the argument if you have osteoporosis while doing weight-bearing exercices)
- For (B), who says that "weight-bearing exercices" cannot be a kind of medical therapy? Thus, making (B) an answer that strengthens the argument, therefore making (B) not the correct answer.
I usually understand when I make mistakes. However, if someone could help me realize why I am wrong for (A) and for (B), that would be super helpful.
Cheers from France!
Benjamin